Get Answers to all your Questions

header-bg qa
Filter By

All Questions

A pole 6 m high casts a shadow 2\sqrt{3} m long on the ground, then the Sun’s elevation is

(A) 60°            (B) 45°            (C) 30°            (D) 90°

Answer.          [A]                  
Solution.         Given :
height pole = 6 m 
Shadow of pole = 2\sqrt{3}m
Now make figure according to given condition

Let angle of elevation is \alpha
\therefore \tan \alpha = \frac{Perpendicular}{base} 
\tan \alpha = \frac{6}{2\sqrt{3}}
\tan \alpha = \frac{3}{\sqrt{3}}= \sqrt{3}
\tan \alpha = \tan 60^{\circ}              \left [ \because \tan 60^{\circ}=\sqrt{3} \right ]
a = 60°
Hence the Sun's elevation is 60°.

View Full Answer(1)
Posted by

infoexpert27

sin (45° + θ) – cos (45° – θ) is equal to
(A) 2cosθ        (B) 0                (C) 2sinθ         (D) 1

Answer.     [B]                  
Solution.    Here

:\sin \left ( 45^{\circ}+\theta \right )-\cos \left ( 45^{\circ}-\theta \right )
  Sin[90° - (45°- θ)] – cos(45°- θ)  
\left [ \because \left ( 45^{\circ} +\theta \right ) = \left ( 90^{\circ}-\left ( 45-\theta \right ) \right )\right ]           
  Cos(45°- θ) – cos(45°- θ)         [\because sin (90 – θ) = cosθ]
= 0
Hence option (B) is correct

View Full Answer(1)
Posted by

infoexpert27

Crack CUET with india's "Best Teachers"

  • HD Video Lectures
  • Unlimited Mock Tests
  • Faculty Support
cuet_ads

If sinθ – cosθ = 0, then the value of (sin4θ + cos4θ) is

(A) 1                (B)3/4                     (C) 1/2                   (D) 1/4

\sin\theta-\cos\theta=0
squaring both sides we get
(\sin\theta-\cos\theta)^2=0
\sin^2\theta+\cos^2\theta-2\sin\theta\cos\theta=0
(\therefore (a – b)2 = a2 + b2 – 2ab)
\sin^2\theta+\cos^2\theta=2\sin\theta\cos\theta.......(1)
1=2\sin\theta\cos\theta\because (\sin^2\theta+\cos^2\theta=1)
\frac{1}{2}=\sin \theta \cos \theta
Squaring both sides we get

\frac{1}{4}=\sin^{2} \theta \cos^{2} \theta       …(2)
Now squaring both side of equation (1) we get
\left ( \sin ^{2}\theta +\cos ^{2}\theta \right )^{2}= \left ( 2\sin \theta \cos \theta \right )^{2}
\left ( \sin ^{2}\theta \right )^{2}+\left ( \cos ^{2}\theta \right )^{2}+2\sin ^{2}\theta \cdot \cos ^{2}\theta = 4\sin ^{2}\theta \cos ^{2}\theta
\left [ \because \left ( a+b \right )^{2} = a^{2}+b^{2}+2ab\right ]
\left ( \sin ^{4}\theta \right )+\left ( \cos ^{4}\theta \right )= 4\sin ^{2}\theta \cos ^{2}\theta -2\sin ^{2}\theta \cos ^{2}\theta
\sin ^{4}\theta +\cos ^{4}\theta = 2\sin ^{2}\theta \cos ^{2}\theta
  (Use equation (2))
\sin ^{4}\theta +\cos ^{4}\theta = 2\left ( \frac{1}{4} \right )
\sin ^{4}\theta +\cos ^{4}\theta =\frac{1}{2}
Hence option (C) is correct.

View Full Answer(1)
Posted by

infoexpert27

If  4 \tan\theta= 3 then  \left ( \frac{4\sin \theta -\cos \theta }{4\sin \theta +\cos \theta } \right )  is equal to

(A)\frac{2}{3}                     (B)\frac{1}{3}                     (C)\frac{1}{2}                (D)\frac{3}{4}


Hence option (C) is correct.

\\\left ( \frac{4\sin \theta -\cos \theta }{4\sin \theta +\cos \theta } \right )\text{ Divide numerator and denominator by and rewriting the given expression}\\\\\Rightarrow ( \frac{4\sin \theta -\cos \theta }{4\sin \theta +\cos \theta })=\frac{4\frac{\sin\theta}{cos\theta}-\frac{cos\theta}{cos\theta}}{4\frac{\sin\theta}{cos\theta}+\frac{cos\theta}{cos\theta}}\\\\=\frac{4\tan\theta-1}{4\tan\theta+1}=\frac{3-1}{3+1}=\frac{2}{4}=\frac{1}{2}\ (\text{Given }4\tan\theta=3)

View Full Answer(1)
Posted by

infoexpert27

JEE Main high-scoring chapters and topics

Study 40% syllabus and score up to 100% marks in JEE

The value of the expression
\left [ \frac{\sin ^{2}22^{\circ}+\sin ^{2}68^{\circ}}{\cos ^{2}22^{\circ}+\cos ^{2}68^{\circ}}+\sin ^{2}63^{\circ}+\cos ^{2}63^{\circ}\sin 27^{\circ} \right ]\ is

(A) 3                (B) 2                (C) 1                (D) 0

Answer.          [B]                  
Solution.

\left [ \frac{\sin ^{2}22^{\circ}+\sin ^{2}68^{\circ}}{\cos ^{2}22^{\circ}+\cos ^{2}68^{\circ}}+\sin ^{2}63^{\circ}+\cos ^{2}63^{\circ}\sin 27^{\circ} \right ]
\frac{\sin ^{2}22^{\circ}+\sin ^{2}68^{\circ}}{\cos ^{2}22^{\circ}+\cos ^{2}68^{\circ}}+\sin ^{2}63^{\circ}+\cos ^{2}63^{\circ}\sin \left ( 90^{\circ}-63^{\circ} \right )
\left [ \because \sin \left ( 90^{\circ}-\theta \right )= \cos \theta \right ]
\frac{\sin ^{2}22^{\circ}+\sin ^{2}68^{\circ}}{\cos ^{2}22^{\circ}+\cos ^{2}68^{\circ}}+\sin ^{2}63^{\circ}+\cos 63^{\circ}\times \cos 63^{\circ}
= \frac{\sin ^{2}22^{\circ}+\sin ^{2}\left (90^{\circ}-22^{\circ} \right )}{\cos ^{2}\left ( 90^{\circ}-68^{\circ} \right )+\cos ^{2}68^{\circ}}+\sin ^{2}63^{\circ}+\cos ^{2}63^{\circ}
\begin{Bmatrix} \because 68^{\circ}= \left ( 90^{\circ}-22^{\circ} \right ) & \\ 22^{\circ}= \left ( 90^{\circ}-68^{\circ} \right )& \end{Bmatrix}
= \frac{\sin ^{2}22^{\circ}+\cos ^{2}22^{\circ}}{\sin ^{2}68^{\circ}+\cos ^{2}68^{\circ}}+\sin ^{2}63^{\circ}+\cos ^{2}63^{\circ}
\begin{Bmatrix} \because \sin \left ( 90^{\circ}-\theta \right ) = \cos \theta & \\ \cos \left ( 90^{\circ} -\theta \right )= \sin \theta & \end{Bmatrix}
= \frac{1}{1}+1\; \left [ \because \sin ^{2}\theta +\cos ^{2}\theta = 1 \right ]
= 1+ 1 =2
Hence option (B) is correct.

View Full Answer(1)
Posted by

infoexpert27

Given that sinα =1/2 and cosβ =1/2 , then the value of (α + β) is

(A) 0°              (B) 30°            (C) 60°            (D) 90°

Answer.      [D]      
Solution.   

\\\sin \alpha=1/2\Rightarrow \alpha=30^0\\\cos \beta=1/2\Rightarrow \beta=60^0\\\therefore \alpha+\beta=90^0
Hence option (D) is correct.

 

View Full Answer(1)
Posted by

infoexpert27

NEET 2024 Most scoring concepts

    Just Study 32% of the NEET syllabus and Score up to 100% marks


If sinA + sin2A = 1, then the value of the expression (cos2A+ cos4A) is

(A) 1                (B)1/2                (C) 2                (D) 3

Answer.     [A]      

Solution.   It is given that sinA + sin2A = 1          …(*)
sinA = 1 – sin2A
sinA = cos2A               …(1)       ( \because 1 – sin2A = cos2A)
Squaring both sides we get
sin2 A = cos4A             …(2)
Hence cos2A + cos4A =
= sinA + sin2A            {using (1) and (2)}
= sinA + sin2A = 1      (given)
Hence option (A) is correct.

 

View Full Answer(1)
Posted by

infoexpert27

If ΔABC is right angled at C, then the value of cos (A+B) is

(A) 0 \ \ \ \ \ (B) 1 \ \ \ \ \ (C)1/2 \ \ \ \ \ (D)\frac{\sqrt{3}}{2}                 

Answer.     [A]                  
Solution.     It is given that \angleC = 90°
               

In \bigtriangleupABC

\angleA +\angleB +\angleC = 180             [\because  sum of interior angles of triangle is 180°]
\angleA + \angleB + 90o = 180            [\because C = 90° (given)]
\angleA + \angleB = 1800 – 900
\angleA + \angleB = 90°          …(1)
cos(\angleA +\angleB) = cos (90°)
cos(90°) = 0
[\because from the table of trigonometric ratios of angles we know that cos 90° = 0]
Hence option A is correct.

 

View Full Answer(1)
Posted by

infoexpert27

Crack CUET with india's "Best Teachers"

  • HD Video Lectures
  • Unlimited Mock Tests
  • Faculty Support
cuet_ads

If cos 9α = sinα and 9α < 90° , then the value of tan5α is
  (A) \frac{1}{\sqrt{3}}            (B) {\sqrt{3}             (C) 1                (D) 0

Answer.          [C]                  
Solution.         Given :- cos 9\alpha = sin\alpha
cos9 \alpha = cos(90 – \alpha)
  \because (cos (90 – \alpha) = sin\alpha)
  9\alpha = 90 – \alpha
9\alpha+ \alpha = 90
10 \alpha= 90
\alpha = \frac{90}{10}
Now tan 5\alpha is
Put \alpha = 9 we get
tan 5\times (9)
tan 450
= 1
{\because from the table of trigonometric ratios of angles we know that tan 450 = 1}
Hence option C is correct.

View Full Answer(1)
Posted by

infoexpert27

The value of (tan1° tan2° tan3° ... tan89°) is
(A) 0                (B) 1                (C) 2                (D)1/2

Answer.          [B]                  
Solution.         Given :-tan1° tan2° tan3° ... tan89°
tan1° tan2° tan3° ... tan89°tan87° tan 88° tan89°       …(1)
We can also write equation (1) in the form of
[tan (900 – 890) . tan (900 – 880). tan (900 – 870) …… tan 87° . tan 88° tan 89°]
[\because we can write tan 1° in the form of tan (900 – 890) similarly we can write other values]
[cot 890 . cot 880. cot 870 …. tan 870 . tan 880 . tan 890]
\because   [tan (902\theta) = cot\theta ]

Also

\left [ \frac{1}{\tan 89^{\circ}}\frac{1}{\tan 88^{\circ}} \frac{1}{\tan 87^{\circ}}\cdots \tan 87^{\circ}.\tan 88^{\circ}\tan 89^{\circ} \right ]

\because Throughout all terms are cancelled by each other and remaining will be tan45

Hence the value is 1

 \because option B is correct.

View Full Answer(1)
Posted by

infoexpert27

JEE Main high-scoring chapters and topics

Study 40% syllabus and score up to 100% marks in JEE

filter_img